brainliest goes to whoever answers the question correctly also if you want more points answer my other questions

Brainliest Goes To Whoever Answers The Question Correctly Also If You Want More Points Answer My Other

Answers

Answer 1

Answer:

B. y = 85x

Step-by-step explanation:

First, find the rate of change of the graph:

Rate of change = change in y/change in x

Using two points (0, 0) and (4, 300),

Rate of change (m) = (300 - 0)/(4 - 0) = 300/4

Rate of change (m) = 75

Compare this with the rate of change of the given equations in the options.

✔️Option A: y = 65x + 100

The rate of change of this equation is 65, which is less than the rate of change of the given graph (75)

✔️Option B: y = 85x

The rate of change of this equation is 85, which is greater than the rate of change of the given graph (75)

✔️Option B: y = 50x + 150

The rate of change of this equation is 50, which is less than the rate of change of the given graph (75)

✅Therefore, the equation that has a greater rate of change than the one shown in the graph is y = 85x


Related Questions

hello please help i’ll give brainliest

Answers

Answer:

Volcanic Eruptions, Earthquakes, Erosion

I am very sorry if this is wrong

Have a great rest of your day and good luck!

Answer:

the answer is a, b, and d

Step-by-step explanation:

erosion is caused by wearing down overtime

a car is going 40 miles per hour for 3 hours how far did the car go?
can anyone explain to me

Answers

Answer:

120miles

Step-by-step explanation:

I am not sure exactly I just guessed that since it's 40 miles an hour, it would be 40 x 3 ~ but am not sure... hope it works out!

btw am rlly bad at this :'')

- sry if am wrong

                           

7676767676767676767676767

Answers

Answer:

uhhh is this a trick question lol

have a good day :D

Step-by-step explanation:

Someone answer this for me

Answers

Answer:

I think it is B or 17 ft

Step-by-step explanation:

Hope this helps. Tell me if I'm wrong

Answer: Probably 30

Step-by-step explanation: If you multiply 12 x 5 = 60, which we don't see that answer. I'll say C because 12 + 12 is 24. Add 5 and you get 29. The closer answer to 29 is 30. I could be wrong because I'm only in Middle School and kinda get confused with questions like these.

what is the median !!! help!! alegbra 2. marking brainliest

Answers

Answer:

The answer is 8.5

Step-by-step explanation:

Just list all the numbers and cross them out one by one.

Add up all of the numbers and divide by the number of numbers in the data set. The median is the central number of a data set. Arrange data points from smallest to largest and locate the central number. This is the median.

*** 10 points**** if you help me

Answers

Answer:

h squared is 65 but h is 8.06

Step-by-step explanation:

use the pythagorean theorem

7^2+4^2=

49+16= 65

square root of 65 is 8.06

hope this helps

Help! it's easy but I need help

Answers

Answer:

no lol

Step-by-step explanation:

Answer:

i will tell you what to do and then you do the work

Step-by-step explanation:

arrange the numbers from lowest to largest

count the amount of numbers to find how many students wrote each test

to find the mean add all of the numbers up then divide by the amount of numbers there are

to find the mode it that is the number that appears the most or the most common value

to find the median is the middle number, when you put it from lowest to largest, it will be the middle number

hope this helps

how many lines of semmetry does a rectangle have

Answers

Answer:

A rectangle has 2 lines of symmetry

use the matrix tool to solve the system of equations enter the answer as an ordered pair. 8x+5y=9 -x+y=7

Answers

Answer:

x = -44/13

y = -65/13

Step-by-step explanation:

Using matrix form means using the crammers rule

The matrix form of the expression is written as;

[tex]\left[\begin{array}{ccc}8&5\\-1&1\\\end{array}\right] \left[\begin{array}{ccc}x\\y\\\end{array}\right] = \left[\begin{array}{ccc}9\\7\\\end{array}\right][/tex]

AX = B

taking the determinant of A;

|A| = 8(1) - 5(-1)

|A| = 8 + 5

|A| = 13

After replacing the first row with the column matrix;

[tex]A_x =\left[\begin{array}{ccc}9&5\\7&-1\\\end{array}\right][/tex]

|Ax| = 9(-1)-5(7)

||Ax| = -9 - 35

|Ax| = -44

x = |Ax|/|A|

x = -44/13

similarly for y

[tex]A_x =\left[\begin{array}{ccc}8&9\\-1&7\\\end{array}\right][/tex]

|Ay| = 8(7)+9

|Ay| = 56+9

|Ay| = 65

y = |Ay|/|A|

y = -65/13

A 2-quart carton of sour cream costs $4.32. What is the price per cup?

Answers

Answer:

0.54 cents

Step-by-step explanation:

4 cups per quart = 8 cups per 2 quarts

4.32/8 = .54

What are all the zeros of the function f(x) = 4x^2 + 8x + 3?

Answers

Answer: -1/2 and -3/2

Step-by-step explanation:

X1= -3/2 x2= - 1/2

I think I’m not sure

help please please please

Answers

Answer:1152 miles

Step-by-step explanation: we know he can drive 384 in 6 hours so if we multiply 6 by 3 its 18 so we must multiply 384 by 3 and the answer would be 1152

1152 miles would be your answer

can someone help me on this question please

Answers

Answer:

y = -2x + 5 [ 1st option ]

Step-by-step explanation:

Consider any two points , say

(1,3) (2,1)

Firstly let's find the slope of the line as ,

=> m = y₁ - y₂ / x₁ - x₂

=> m = 3-1 / 1-2

=> m = 2/-1

=> m = -2

Third point is, (3 , -1) . Using slope point form .

=> y - y₁ = m ( x - x₁ )

=> y - (-1) = -2( x - 3)

=> y + 1 = -2x +6

=> y +2x +1-6=0

=> y +2x -5 = 0

=> y = -2x + 5

On looking at options first one is correct

Hence the equation of line is y = -2x + 5.

Fill in the blank to complete this trig
ratio.
y
tan a =
a
Х
[?]
B
N
B
С
у
Z

Answers

Answer:

z/y

Step-by-step explanation:

Given the three functions: f(x)=4(2) g(x)=8x² + 24x , and h(x)= 4x + 8
Label each function as quadratic, linear, or exponential.
f(x)=
g(x)=
h(x)=

Answers

Answer:

f(x)=exponential

g(x)= quadratic

h(x)= linear

Step-by-step explanation:

exponential function

[tex] {ab}^{x} [/tex]

Quadratic function

[tex]a {x}^{2} + bx + c[/tex]

Linear function

[tex]y = mx + c[/tex]

Hi there!

[tex] \large \boxed{ y = {a}^{x} \: \: (a \neq 0, \: a > 1)}[/tex]

The equation above is an exponential family type.

[tex] \large \boxed{y = a {x}^{2} + bx + c \: \: (a \neq 0)}[/tex]

The equation above is quadratic type.

[tex] \large \boxed{ y = mx + b}[/tex]

The last type above is a linear type.

Hence, the answers are:

f(x) = exponentialg(x) = quadratich(x) = linear

An angle measures 79.6° less than the measure of its complementary angle. What is the measure of each angle?

Answers

Answer:

y = 259.6/2 = 129.8 degrees. so x = 129.8 - 79.6 = 50.2 degrees.

Step-by-step explanation:

I got this answer from somewhere else. but it is correct. i hope this helped.

Dustin has a spinner that is divided into 5 equal-size sections colored pink, blue, orange, white, and green. What is the probability that Dustin spins pink on the 1st spin and then spins a color other than pink on the 2nd spin?

Answers

Answer:

1/6

Step-by-step explanation:

1st spin:

1 out of the 5 sections are pink, so the probability of spinning a pink on the 1st spin is 1/5.

2nd spin:

5 out of the 6 sections are not pink, so the probability of spinning a pink on the 2nd is 5/6.

Multiply the probabilities of the two spins to get the answer.

[tex]\frac{1}{5} *\frac{5}{6} =\frac{5}{30} =\frac{1}{6}[/tex]

HI PLS HELP ASAP ILL GIVE BRAINLIEST

Answers

Answer:

Monsoon.

Step-by-step explanation:

Answer:

upper level

Step-by-step explanation:

I think it is because it it sounds like it match

What percent df 160 is 56? Use the percent bar model.

Answers

Answer:

35%

i think

Step-by-step explanation:

I will give brainliest to the first person to answer the question! Please be correct and please help <33

Answers

Answer:

112.25

Step-by-step explanation:

Answer:110.25

Step-by-step explanation:

Tony bought a computer. He paid $450.00 plus sales tax. The sales tax
was 6%. How much did he pay in sales tax?

Answers

Answer:

$27

Step-by-step explanation:

We have to find 6% of $450. We can make an expression and solve it:

6/100•450

We can cancel out factors now:

3/50•450

Now we'll cancel out 50 and 450:

3x9=$27

And that's your answer! Hope this helps =)

what is the number 154 798 105 in expanded notation​

Answers


Expanded form:

1 × 100,000,000
+ 5 × 10,000,000
+ 4 × 1,000,000
+ 7 × 100,000
+ 9 × 10,000
+ 8 × 1,000
+ 1 × 100
+ 0 × 10
+ 5 × 1

I hope it helps :)

Slope of the line that passes through (8,-9) and (14,-9)

Answers

Answer:

slope = 0

Step-by-step explanation:

Calculate the slope m using the slope formula

m = [tex]\frac{y_{2}-y_{1} }{x_{2}-x_{1} }[/tex]

with (x₁, y₁ ) = (8, - 9) and (x₂, y₂ ) = (14, - 9)

m = [tex]\frac{-9+9}{14-8}[/tex] = [tex]\frac{0}{6}[/tex] = 0

Forty-five percent of what number is 15.75?​

Answers

45 percent (calculated percentage %) of what number equals 15.75? Answer: 35.

Answer:

35

Step-by-step explanation:

15.75 / 0.45 = 35

Since there are no restrictions on the strings, we can think of this problem as drawing 5 random digits in sequence. There is a 9/10 chance each time we draw to NOT get a 4. Every time we draw, we multiply by 9/10 again until we've draw 5 digits. Thus, the answer is (9/10)^5 :)

Answers

Answer:

[tex]Pr = 0.59049[/tex]

Step-by-step explanation:

See Comment for Complete Question

Given that A has 5 digits. A digit in A can be chosen from 0 to 9 (10 digits).

However, selection without 4 means that we can only select from 0 to 3 and 5 to 9 (altogether, that is 9 digits)

The probability of selecting a number other than 4 in each of the digit is:

[tex]1st \to \frac{9}{10}[/tex]

[tex]2nd \to \frac{9}{10}[/tex]

[tex]3rd \to \frac{9}{10}[/tex]

[tex]4th \to \frac{9}{10}[/tex]

[tex]5th \to \frac{9}{10}[/tex]

So, the required probability is:

[tex]Pr = 1st * 2nd * 3rd * 4th * 5th[/tex]

[tex]Pr = \frac{9}{10} *\frac{9}{10} *\frac{9}{10} *\frac{9}{10} *\frac{9}{10}[/tex]

[tex]Pr = \frac{9^5}{10^5}[/tex]

[tex]Pr = \frac{59049}{100000}[/tex]

[tex]Pr = 0.59049[/tex]

if you roll a dice one time what is the probability for it to land on a number greater than 2​

Answers

Answer:

4/6

Step-by-step explanation:

The answer is 4/6
This is because there is 6 sides to a dice and the side with 1 and 2 are the only sides with numbers less than 2. So only 4 sides out of the 6 can be greater giving the answer 4/6!

i will give brainiest if correct please hurry!!
You flip a coin 50 times and get 26 tails. What is the experimental probability of flipping heads? Write your answer as a simplified fraction or a percentage.

Answers

Answer:

12/25

Step-by-step explanation:

If you got 26 tails, that means you got 50 - 26 or 24 heads. The experimental  probability can be represented by the desirable flips over the total number of flips. We are looking for heads, so the amount of heads is the amount of desirable flips, and the total is 50. From this, we get the fraction 24/50 which simplifies to 12/25.

Give the domain.

y =2 / (5x - 4)(4x - 9)

Answers

Answer:

x ≠ 4/5, x ≠ 9/4

You can also write it as "all real numbers except 4/5 and 9/4."

Step-by-step explanation:

The denominator of a fraction cannot equal 0. We can write this relation to find what x does NOT equal:

(5x - 4)(4x - 9) ≠ 0

We can use the zero-product property to get two separate equations:

5x - 4 ≠ 0

5x ≠ 4

x ≠ 4/5

4x - 9 ≠ 0

4x ≠ 9

x ≠ 9/4

The answer above me is correct

Jay wants to spend less than $80 He plans to buy a pair of jeans that cost $19.99 and some shirts that cost \$14.99 each Write an equation or inequality to represent the number of shirts Jay can buy Define all variables used • What is the greatest number of shirts that Jay can buy while spending less than $80 ? Show your work​

Answers

Answer:

give me brainllest if it is right

a. $19.99 + $14.99 = $34.98

b. Jay can buy 5 shirts without spending more than $80

Step-by-step explanation:

Select the correct answer. What is the approximate distance between points D and E? Round your answer to the nearest hundredth. A.

Answers

Answer:

The approximate distance between points D and E is 8.25

Step-by-step explanation:

The figure is as follows :

To find - What is the approximate distance between points D and E ?

Solution -

By Pythagoras Theorem, we get

(DE)² = (FE)² + (DF)²

        = (8)² + (2)²

        = 64 + 4

        = 68

⇒(DE)² = 68

⇒DE = √68

⇒DE = 8.246

∴ we get

The approximate distance between points D and E is 8.25

Other Questions
can someone please help me Individuals in a population of purple people earters are polymorphic at a single locus that leads to variation in purple people eater pup viability. 45% of the pups with genotype PP are viable, 54% of pups with genotype Pp are viable, but only 4% of pups with genotype pp are viable. What model of natural selection best fits these data If 50 grams of sweats cost $2.10,find the cost of 380 of sweats,giving your answer correct to the nearest 5 cent The graph of a linear function is shown.A coordinate plane with a straight line passing through (negative 4, 2), (0, 0), and (4, negative 2).Which word describes the slope of the line?positivenegativezeroundefined Who found the printing press 4 Why will alcohol keep on boiling even after the flame was turned down? Solve for x.2x + 243x - 26x = [?] what are 2 or 3 Second Step skills or strategies youve used or plan to use in your life?(THIS IS REALLY FOR ADVISEARY) *NO LINKS* Figure abc is a triangle with a base of 22 inches if abc is dilated by a scale factor of 0.5 what will be the base of the figure How are pickles made? Find the 8th term 50,20,8 atleast, had, we, ten minutes, wait, for, to. arrange the sentence The following list shows three factors that can either increase or decrease the probability of a Type II error. The sample size is increased. The significance level is increased. The standard error is increased. Which factors alone will cause the probability of a Type II error to increase? Can someone help me out on all three questions Id appreciate ittt Jess says that two of the three points below are on a line through the origin.(3.8), (6, 21), and (9, 24)Part AWhich two points could lie on the same line through the origin? Explain why. Explain whythe third point could not.* I will give brainliest* Write the equation of the line in slope-intercept form that has a slope of -1/2 and goes through the point (2,3) im having a hard time trying to figure out which words are llanas. pls help Priya, variable "p", jumped 3 inches more than Xeno, x, in a high jump contest. Mai, m, jumped three times as many inches higher than Priya. If Mai jumped 14 inches, how many inches did Xeno reach? (You can use a tape diagram or hanger diagram to help solve this problem). ***EXPLAIN how you found your answer below: Name a point that is not contained in any of the 3 lines a, b, or c What is the greatestcommon factor (GCF)of 25 and 75?